Let $G$ be a non-abelian finite $p$-group, do we have $Z(G) leq Phi(G)$ in general?












0














Let $G$ be a non-abelian finite $p$-group. I wonder if $Z(G) leq Phi(G)$, where $Phi(G)$ is the Frattini subgroup of $G$? I'd known that it is true for non-abelian groups of order $p^3$ but I doubt it is true in general or not.



I understand that it isn't true in general. As showed below it isn't true for some group of order $p^4$.










share|cite|improve this question
























  • @Yasmin: Your edit does not address the original problem: why did you think it was true, or why did you want to prove it? As to what you write now, you “ask” whether it is true, and then you say you “understand” it isn’t. That makes the question self-contradictory.
    – Arturo Magidin
    2 days ago










  • @ArturoMagidin Would you please suggest how to edit it in a right way?
    – Yasmin
    2 days ago










  • @Yasmin: Add context. Add why you were interested in this; why you thought it might be true. Do not falsify the record by writing nonsense after the fact when it turned out that what you wanted to prove is actually false. That would be a good place to start, instead of asking people to do your work for you, again.
    – Arturo Magidin
    2 days ago






  • 2




    The general point is that if you do not know whether something is true or not, then you should not say "help me prove this", you should say "help me decide whether this is true". Otherwise people might waste time trying to prove something that isn't true. You should also say where the problem comes from: is it a homework exercise, is it a problem from a book, is it your own problem? (Occasionally problems from books are wrong, and of course in that case everybody would be confused!)
    – Derek Holt
    2 days ago








  • 1




    I have voted to reopen, because the question itself has some interest and it has been correctly answered.
    – Derek Holt
    2 days ago
















0














Let $G$ be a non-abelian finite $p$-group. I wonder if $Z(G) leq Phi(G)$, where $Phi(G)$ is the Frattini subgroup of $G$? I'd known that it is true for non-abelian groups of order $p^3$ but I doubt it is true in general or not.



I understand that it isn't true in general. As showed below it isn't true for some group of order $p^4$.










share|cite|improve this question
























  • @Yasmin: Your edit does not address the original problem: why did you think it was true, or why did you want to prove it? As to what you write now, you “ask” whether it is true, and then you say you “understand” it isn’t. That makes the question self-contradictory.
    – Arturo Magidin
    2 days ago










  • @ArturoMagidin Would you please suggest how to edit it in a right way?
    – Yasmin
    2 days ago










  • @Yasmin: Add context. Add why you were interested in this; why you thought it might be true. Do not falsify the record by writing nonsense after the fact when it turned out that what you wanted to prove is actually false. That would be a good place to start, instead of asking people to do your work for you, again.
    – Arturo Magidin
    2 days ago






  • 2




    The general point is that if you do not know whether something is true or not, then you should not say "help me prove this", you should say "help me decide whether this is true". Otherwise people might waste time trying to prove something that isn't true. You should also say where the problem comes from: is it a homework exercise, is it a problem from a book, is it your own problem? (Occasionally problems from books are wrong, and of course in that case everybody would be confused!)
    – Derek Holt
    2 days ago








  • 1




    I have voted to reopen, because the question itself has some interest and it has been correctly answered.
    – Derek Holt
    2 days ago














0












0








0


1





Let $G$ be a non-abelian finite $p$-group. I wonder if $Z(G) leq Phi(G)$, where $Phi(G)$ is the Frattini subgroup of $G$? I'd known that it is true for non-abelian groups of order $p^3$ but I doubt it is true in general or not.



I understand that it isn't true in general. As showed below it isn't true for some group of order $p^4$.










share|cite|improve this question















Let $G$ be a non-abelian finite $p$-group. I wonder if $Z(G) leq Phi(G)$, where $Phi(G)$ is the Frattini subgroup of $G$? I'd known that it is true for non-abelian groups of order $p^3$ but I doubt it is true in general or not.



I understand that it isn't true in general. As showed below it isn't true for some group of order $p^4$.







group-theory finite-groups normal-subgroups p-groups maximal-subgroup






share|cite|improve this question















share|cite|improve this question













share|cite|improve this question




share|cite|improve this question








edited 2 days ago

























asked 2 days ago









Yasmin

13511




13511












  • @Yasmin: Your edit does not address the original problem: why did you think it was true, or why did you want to prove it? As to what you write now, you “ask” whether it is true, and then you say you “understand” it isn’t. That makes the question self-contradictory.
    – Arturo Magidin
    2 days ago










  • @ArturoMagidin Would you please suggest how to edit it in a right way?
    – Yasmin
    2 days ago










  • @Yasmin: Add context. Add why you were interested in this; why you thought it might be true. Do not falsify the record by writing nonsense after the fact when it turned out that what you wanted to prove is actually false. That would be a good place to start, instead of asking people to do your work for you, again.
    – Arturo Magidin
    2 days ago






  • 2




    The general point is that if you do not know whether something is true or not, then you should not say "help me prove this", you should say "help me decide whether this is true". Otherwise people might waste time trying to prove something that isn't true. You should also say where the problem comes from: is it a homework exercise, is it a problem from a book, is it your own problem? (Occasionally problems from books are wrong, and of course in that case everybody would be confused!)
    – Derek Holt
    2 days ago








  • 1




    I have voted to reopen, because the question itself has some interest and it has been correctly answered.
    – Derek Holt
    2 days ago


















  • @Yasmin: Your edit does not address the original problem: why did you think it was true, or why did you want to prove it? As to what you write now, you “ask” whether it is true, and then you say you “understand” it isn’t. That makes the question self-contradictory.
    – Arturo Magidin
    2 days ago










  • @ArturoMagidin Would you please suggest how to edit it in a right way?
    – Yasmin
    2 days ago










  • @Yasmin: Add context. Add why you were interested in this; why you thought it might be true. Do not falsify the record by writing nonsense after the fact when it turned out that what you wanted to prove is actually false. That would be a good place to start, instead of asking people to do your work for you, again.
    – Arturo Magidin
    2 days ago






  • 2




    The general point is that if you do not know whether something is true or not, then you should not say "help me prove this", you should say "help me decide whether this is true". Otherwise people might waste time trying to prove something that isn't true. You should also say where the problem comes from: is it a homework exercise, is it a problem from a book, is it your own problem? (Occasionally problems from books are wrong, and of course in that case everybody would be confused!)
    – Derek Holt
    2 days ago








  • 1




    I have voted to reopen, because the question itself has some interest and it has been correctly answered.
    – Derek Holt
    2 days ago
















@Yasmin: Your edit does not address the original problem: why did you think it was true, or why did you want to prove it? As to what you write now, you “ask” whether it is true, and then you say you “understand” it isn’t. That makes the question self-contradictory.
– Arturo Magidin
2 days ago




@Yasmin: Your edit does not address the original problem: why did you think it was true, or why did you want to prove it? As to what you write now, you “ask” whether it is true, and then you say you “understand” it isn’t. That makes the question self-contradictory.
– Arturo Magidin
2 days ago












@ArturoMagidin Would you please suggest how to edit it in a right way?
– Yasmin
2 days ago




@ArturoMagidin Would you please suggest how to edit it in a right way?
– Yasmin
2 days ago












@Yasmin: Add context. Add why you were interested in this; why you thought it might be true. Do not falsify the record by writing nonsense after the fact when it turned out that what you wanted to prove is actually false. That would be a good place to start, instead of asking people to do your work for you, again.
– Arturo Magidin
2 days ago




@Yasmin: Add context. Add why you were interested in this; why you thought it might be true. Do not falsify the record by writing nonsense after the fact when it turned out that what you wanted to prove is actually false. That would be a good place to start, instead of asking people to do your work for you, again.
– Arturo Magidin
2 days ago




2




2




The general point is that if you do not know whether something is true or not, then you should not say "help me prove this", you should say "help me decide whether this is true". Otherwise people might waste time trying to prove something that isn't true. You should also say where the problem comes from: is it a homework exercise, is it a problem from a book, is it your own problem? (Occasionally problems from books are wrong, and of course in that case everybody would be confused!)
– Derek Holt
2 days ago






The general point is that if you do not know whether something is true or not, then you should not say "help me prove this", you should say "help me decide whether this is true". Otherwise people might waste time trying to prove something that isn't true. You should also say where the problem comes from: is it a homework exercise, is it a problem from a book, is it your own problem? (Occasionally problems from books are wrong, and of course in that case everybody would be confused!)
– Derek Holt
2 days ago






1




1




I have voted to reopen, because the question itself has some interest and it has been correctly answered.
– Derek Holt
2 days ago




I have voted to reopen, because the question itself has some interest and it has been correctly answered.
– Derek Holt
2 days ago










1 Answer
1






active

oldest

votes


















7














Let $H$ be the nonabelian group of order $p^3$ and exponent $p$. Let $C_p$ be a cyclic group of order $p$. Let $G=Htimes C_p$. Then $Z(G)$ is a direct product of two copies of $C_p$, one contained in $H$, the other the second direct factor of $G$.



However, $H$ is maximal in $G$, and so $Phi(G)subseteq H$. Thus, $Z(G)$ is not contained in $Phi(G)$.



More generally, given any nonabelian $p$-group $K$, letting $G=Ktimes C_p$ gives you a group with center $Z(G) = Z(K)times C_p$, and with $K$ maximal and hence $Phi(G)subseteq K$, hence a counterexample to the assertion to $Z(G)subseteq Phi(G)$.






share|cite|improve this answer



















  • 1




    Thank you so much.
    – Yasmin
    2 days ago











Your Answer





StackExchange.ifUsing("editor", function () {
return StackExchange.using("mathjaxEditing", function () {
StackExchange.MarkdownEditor.creationCallbacks.add(function (editor, postfix) {
StackExchange.mathjaxEditing.prepareWmdForMathJax(editor, postfix, [["$", "$"], ["\\(","\\)"]]);
});
});
}, "mathjax-editing");

StackExchange.ready(function() {
var channelOptions = {
tags: "".split(" "),
id: "69"
};
initTagRenderer("".split(" "), "".split(" "), channelOptions);

StackExchange.using("externalEditor", function() {
// Have to fire editor after snippets, if snippets enabled
if (StackExchange.settings.snippets.snippetsEnabled) {
StackExchange.using("snippets", function() {
createEditor();
});
}
else {
createEditor();
}
});

function createEditor() {
StackExchange.prepareEditor({
heartbeatType: 'answer',
autoActivateHeartbeat: false,
convertImagesToLinks: true,
noModals: true,
showLowRepImageUploadWarning: true,
reputationToPostImages: 10,
bindNavPrevention: true,
postfix: "",
imageUploader: {
brandingHtml: "Powered by u003ca class="icon-imgur-white" href="https://imgur.com/"u003eu003c/au003e",
contentPolicyHtml: "User contributions licensed under u003ca href="https://creativecommons.org/licenses/by-sa/3.0/"u003ecc by-sa 3.0 with attribution requiredu003c/au003e u003ca href="https://stackoverflow.com/legal/content-policy"u003e(content policy)u003c/au003e",
allowUrls: true
},
noCode: true, onDemand: true,
discardSelector: ".discard-answer"
,immediatelyShowMarkdownHelp:true
});


}
});














draft saved

draft discarded


















StackExchange.ready(
function () {
StackExchange.openid.initPostLogin('.new-post-login', 'https%3a%2f%2fmath.stackexchange.com%2fquestions%2f3060920%2flet-g-be-a-non-abelian-finite-p-group-do-we-have-zg-leq-phig-in-gen%23new-answer', 'question_page');
}
);

Post as a guest















Required, but never shown

























1 Answer
1






active

oldest

votes








1 Answer
1






active

oldest

votes









active

oldest

votes






active

oldest

votes









7














Let $H$ be the nonabelian group of order $p^3$ and exponent $p$. Let $C_p$ be a cyclic group of order $p$. Let $G=Htimes C_p$. Then $Z(G)$ is a direct product of two copies of $C_p$, one contained in $H$, the other the second direct factor of $G$.



However, $H$ is maximal in $G$, and so $Phi(G)subseteq H$. Thus, $Z(G)$ is not contained in $Phi(G)$.



More generally, given any nonabelian $p$-group $K$, letting $G=Ktimes C_p$ gives you a group with center $Z(G) = Z(K)times C_p$, and with $K$ maximal and hence $Phi(G)subseteq K$, hence a counterexample to the assertion to $Z(G)subseteq Phi(G)$.






share|cite|improve this answer



















  • 1




    Thank you so much.
    – Yasmin
    2 days ago
















7














Let $H$ be the nonabelian group of order $p^3$ and exponent $p$. Let $C_p$ be a cyclic group of order $p$. Let $G=Htimes C_p$. Then $Z(G)$ is a direct product of two copies of $C_p$, one contained in $H$, the other the second direct factor of $G$.



However, $H$ is maximal in $G$, and so $Phi(G)subseteq H$. Thus, $Z(G)$ is not contained in $Phi(G)$.



More generally, given any nonabelian $p$-group $K$, letting $G=Ktimes C_p$ gives you a group with center $Z(G) = Z(K)times C_p$, and with $K$ maximal and hence $Phi(G)subseteq K$, hence a counterexample to the assertion to $Z(G)subseteq Phi(G)$.






share|cite|improve this answer



















  • 1




    Thank you so much.
    – Yasmin
    2 days ago














7












7








7






Let $H$ be the nonabelian group of order $p^3$ and exponent $p$. Let $C_p$ be a cyclic group of order $p$. Let $G=Htimes C_p$. Then $Z(G)$ is a direct product of two copies of $C_p$, one contained in $H$, the other the second direct factor of $G$.



However, $H$ is maximal in $G$, and so $Phi(G)subseteq H$. Thus, $Z(G)$ is not contained in $Phi(G)$.



More generally, given any nonabelian $p$-group $K$, letting $G=Ktimes C_p$ gives you a group with center $Z(G) = Z(K)times C_p$, and with $K$ maximal and hence $Phi(G)subseteq K$, hence a counterexample to the assertion to $Z(G)subseteq Phi(G)$.






share|cite|improve this answer














Let $H$ be the nonabelian group of order $p^3$ and exponent $p$. Let $C_p$ be a cyclic group of order $p$. Let $G=Htimes C_p$. Then $Z(G)$ is a direct product of two copies of $C_p$, one contained in $H$, the other the second direct factor of $G$.



However, $H$ is maximal in $G$, and so $Phi(G)subseteq H$. Thus, $Z(G)$ is not contained in $Phi(G)$.



More generally, given any nonabelian $p$-group $K$, letting $G=Ktimes C_p$ gives you a group with center $Z(G) = Z(K)times C_p$, and with $K$ maximal and hence $Phi(G)subseteq K$, hence a counterexample to the assertion to $Z(G)subseteq Phi(G)$.







share|cite|improve this answer














share|cite|improve this answer



share|cite|improve this answer








edited 2 days ago

























answered 2 days ago









Arturo Magidin

261k32585906




261k32585906








  • 1




    Thank you so much.
    – Yasmin
    2 days ago














  • 1




    Thank you so much.
    – Yasmin
    2 days ago








1




1




Thank you so much.
– Yasmin
2 days ago




Thank you so much.
– Yasmin
2 days ago


















draft saved

draft discarded




















































Thanks for contributing an answer to Mathematics Stack Exchange!


  • Please be sure to answer the question. Provide details and share your research!

But avoid



  • Asking for help, clarification, or responding to other answers.

  • Making statements based on opinion; back them up with references or personal experience.


Use MathJax to format equations. MathJax reference.


To learn more, see our tips on writing great answers.





Some of your past answers have not been well-received, and you're in danger of being blocked from answering.


Please pay close attention to the following guidance:


  • Please be sure to answer the question. Provide details and share your research!

But avoid



  • Asking for help, clarification, or responding to other answers.

  • Making statements based on opinion; back them up with references or personal experience.


To learn more, see our tips on writing great answers.




draft saved


draft discarded














StackExchange.ready(
function () {
StackExchange.openid.initPostLogin('.new-post-login', 'https%3a%2f%2fmath.stackexchange.com%2fquestions%2f3060920%2flet-g-be-a-non-abelian-finite-p-group-do-we-have-zg-leq-phig-in-gen%23new-answer', 'question_page');
}
);

Post as a guest















Required, but never shown





















































Required, but never shown














Required, but never shown












Required, but never shown







Required, but never shown

































Required, but never shown














Required, but never shown












Required, but never shown







Required, but never shown







Popular posts from this blog

Mario Kart Wii

What does “Dominus providebit” mean?

Antonio Litta Visconti Arese